If w + x < 0, is w – y > 0? (1) x + y < 0 (2

This topic has expert replies
User avatar
Legendary Member
Posts: 1556
Joined: Tue Aug 14, 2012 11:18 pm
Thanked: 448 times
Followed by:34 members
GMAT Score:650
If w + x < 0, is w - y > 0?
(1) x + y < 0
(2) y < x < w

I am not sure of the Source of this Question. The OA is B
But, I am getting the answer is D

My Steps to Solutions:
w + x < 0
w + x < y - y
w - y < - y - x
w - y < - (x + y)

Statement 1 says x + y < 0 ... So, w - y > 0 ... Sufficient
Statement 2 says y < x < w ... So,
Either w is +ve , x is - ve hence y is also -ve
or w,x,y all are negative.. in both the cases w-y >0 ... Sufficient
So,[spoiler] [D] [/spoiler]..
BUT, OA is different.
Please help

User avatar
Legendary Member
Posts: 643
Joined: Wed Aug 14, 2013 4:27 am
Thanked: 48 times
Followed by:7 members

by vinay1983 » Mon Sep 16, 2013 11:13 pm
According to me, the question stem can be made as is W>y

So,

Statement 1

No relation given between w and y Not sufficient

Statement 2

y < x < w
Implies y < w

So we can answer our revised question with this, Sufficient

B to me. I hope I am correct.
You can, for example never foretell what any one man will do, but you can say with precision what an average number will be up to!

User avatar
Legendary Member
Posts: 1556
Joined: Tue Aug 14, 2012 11:18 pm
Thanked: 448 times
Followed by:34 members
GMAT Score:650

by theCodeToGMAT » Mon Sep 16, 2013 11:34 pm
Yeah Vinay, the way you solved the question is correct; Even I did that method once I was stuck with the first method I applied to solve it (Method is stated below the question).

The problem is .. I am not able to catch the problem in my First Method.

The approach you applied does gives ambiguous values for "W" ranging from Positive Values 1 to 4 to any negative value. Hence, Statement 1 is Insufficient.
vinay1983 wrote:According to me, the question stem can be made as is W>y

So,

Statement 1

No relation given between w and y Not sufficient

Statement 2

y < x < w
Implies y < w

So we can answer our revised question with this, Sufficient

B to me. I hope I am correct.

User avatar
GMAT Instructor
Posts: 3650
Joined: Wed Jan 21, 2009 4:27 am
Location: India
Thanked: 267 times
Followed by:80 members
GMAT Score:760

by sanju09 » Tue Sep 17, 2013 1:18 am
rahulmittal87 wrote:If w + x < 0, is w - y > 0?
(1) x + y < 0
(2) y < x < w

I am not sure of the Source of this Question. The OA is B
But, I am getting the answer is D

My Steps to Solutions:
w + x < 0
w + x < y - y
w - y < - y - x
w - y < - (x + y)

Statement 1 says x + y < 0 ... So, w - y > 0 ... Sufficient
Statement 2 says y < x < w ... So,
Either w is +ve , x is - ve hence y is also -ve
or w,x,y all are negative.. in both the cases w-y >0 ... Sufficient
So,[spoiler] [D] [/spoiler]..
BUT, OA is different.
Please help
Hi rahulmittal87,

I. On GMAT, relying too much on algebraic approaches can sometimes make us overlook few ground facts that may be some hidden constraints throughout the question. In your pre-work, the introduction of y took place accidentally, simply because of its unpredictable sign and size in compare to w and x, which can change the inequality altogether. When nothing had been mentioned about y so far, how can we really play with it inside a risky inequality? This can be better understood by picking suitable set of easy numbers for w, x, and y, and realize the liberty to have so many different suitable set of easy numbers and weird verities for w, x, and y that can change the meaning of the inequality.

Hence, statement 1 is not sufficient.
The mind is everything. What you think you become. -Lord Buddha



Sanjeev K Saxena
Quantitative Instructor
The Princeton Review - Manya Abroad
Lucknow-226001

www.manyagroup.com

Newbie | Next Rank: 10 Posts
Posts: 1
Joined: Fri Oct 26, 2012 12:43 am

by phsr » Tue Sep 17, 2013 1:23 am
rahulmittal87 wrote:If w + x < 0, is w - y > 0?
(1) x + y < 0
(2) y < x < w

I am not sure of the Source of this Question. The OA is B
But, I am getting the answer is D

My Steps to Solutions:
w + x < 0
w + x < y - y
w - y < - y - x
w - y < - (x + y)

Statement 1 says x + y < 0 ... So, w - y > 0 ... Sufficient
Statement 2 says y < x < w ... So,
Either w is +ve , x is - ve hence y is also -ve
or w,x,y all are negative.. in both the cases w-y >0 ... Sufficient
So,[spoiler] [D] [/spoiler]..
BUT, OA is different.
Please help
------------------------------------------------------------------------------
why A is not insufficient is that ...x + w < 0 means x or w is negative or both are so.

However, with only this 2 fact
x + y < 0 (x or y is negative or both) , x + w < 0 (x or w is negative or both)

it's is impossible to know whether x is greater than y.(w - y > 0)

So A is insuffient..

User avatar
Legendary Member
Posts: 1556
Joined: Tue Aug 14, 2012 11:18 pm
Thanked: 448 times
Followed by:34 members
GMAT Score:650

by theCodeToGMAT » Tue Sep 17, 2013 2:14 am
Yeah i know this fact; Actually, I was looking for possible problem in the method which i had used at the first place. I tried to solve the question using the same old algebric tricks in which we try to introduce some variable/value without affecting the balance between LHS & RHS. I wrote "0" as "y-y", which should always be satisfied for any possible value of Y.

Anyways, thanks Sanju for the explanation.


phsr wrote:
rahulmittal87 wrote:If w + x < 0, is w - y > 0?
(1) x + y < 0
(2) y < x < w

I am not sure of the Source of this Question. The OA is B
But, I am getting the answer is D

My Steps to Solutions:
w + x < 0
w + x < y - y
w - y < - y - x
w - y < - (x + y)

Statement 1 says x + y < 0 ... So, w - y > 0 ... Sufficient
Statement 2 says y < x < w ... So,
Either w is +ve , x is - ve hence y is also -ve
or w,x,y all are negative.. in both the cases w-y >0 ... Sufficient
So,[spoiler] [D] [/spoiler]..
BUT, OA is different.
Please help
------------------------------------------------------------------------------
why A is not insufficient is that ...x + w < 0 means x or w is negative or both are so.

However, with only this 2 fact
x + y < 0 (x or y is negative or both) , x + w < 0 (x or w is negative or both)

it's is impossible to know whether x is greater than y.(w - y > 0)

So A is insuffient..

User avatar
GMAT Instructor
Posts: 15539
Joined: Tue May 25, 2010 12:04 pm
Location: New York, NY
Thanked: 13060 times
Followed by:1906 members
GMAT Score:790

by GMATGuruNY » Tue Sep 17, 2013 4:53 am
rahulmittal87 wrote:If w + x < 0, is w - y > 0?
(1) x + y < 0
(2) y < x < w
Question rephrased: Is w > y?

Statement 1: x+y < 0.
Rewritten, y < -x.
Rewritten, w+x < 0 becomes w < -x.
Now we can see that both w and y are less than -x, but no information is given about whether w>y.
Insufficient.

Statement 2: y < x < w.
Thus, y < w.
Sufficient.

The correct answer is B.
Private tutor exclusively for the GMAT and GRE, with over 20 years of experience.
Followed here and elsewhere by over 1900 test-takers.
I have worked with students based in the US, Australia, Taiwan, China, Tajikistan, Kuwait, Saudi Arabia -- a long list of countries.
My students have been admitted to HBS, CBS, Tuck, Yale, Stern, Fuqua -- a long list of top programs.

As a tutor, I don't simply teach you how I would approach problems.
I unlock the best way for YOU to solve problems.

For more information, please email me (Mitch Hunt) at [email protected].
Student Review #1
Student Review #2
Student Review #3

Legendary Member
Posts: 641
Joined: Tue Feb 14, 2012 3:52 pm
Thanked: 11 times
Followed by:8 members

by gmattesttaker2 » Tue Oct 01, 2013 11:29 pm
Hello,

I was wondering why this would be wrong since I thought that we can add/subtract in-equalites as long as they have the same sign:

If w + x < 0, is w - y > 0?
(1) x + y < 0

So, w + x < 0
& y + x < 0

So, w - y < 0


Thanks,
Sri

Master | Next Rank: 500 Posts
Posts: 391
Joined: Sat Mar 02, 2013 5:13 am
Thanked: 50 times
Followed by:4 members

by rakeshd347 » Tue Oct 01, 2013 11:35 pm
theCodeToGMAT wrote:If w + x < 0, is w - y > 0?
(1) x + y < 0
(2) y < x < w

I am not sure of the Source of this Question. The OA is B
But, I am getting the answer is D

My Steps to Solutions:
w + x < 0
w + x < y - y
w - y < - y - x
w - y < - (x + y)

Statement 1 says x + y < 0 ... So, w - y > 0 ... Sufficient
Statement 2 says y < x < w ... So,
Either w is +ve , x is - ve hence y is also -ve
or w,x,y all are negative.. in both the cases w-y >0 ... Sufficient
So,[spoiler] [D] [/spoiler]..
BUT, OA is different.
Please help
Hi Rahul,

To clarify your confusion on adding variable and show you where you went wrong here is my explanation:

w + x < 0
w + x < y - y
w - y < - y - x
w - y < - (x + y)

Now statement 1 says that x+y<0 or -(x+y)>0 we are clear on this.
Now here is what you made the mistake.
-(x+y)>0
-(x+y)>w-y so you concluded that w-y>0 thats wrong.

example:

A>0 ; A>C now you are saying that so C has to be more than 0. no it doesn't have to be true. A could be 5 and C could be -2 or C could be 1 or may be or may be not situation.

Another example:
A>B
A>C here may be the sequence is A>B>C or A>C>B....So two outcomes.

Otherwise when it comes to adding variable to both sides of the inequality it is absolutely fine.

Thanks,
Rakesh

User avatar
Legendary Member
Posts: 1556
Joined: Tue Aug 14, 2012 11:18 pm
Thanked: 448 times
Followed by:34 members
GMAT Score:650

by theCodeToGMAT » Tue Oct 01, 2013 11:51 pm
The problem here is :

I have mistakenly changed the sign in the solution mentioned in the post

w - y < - (x + y)

If x+y<0

that means w - y "<" and NOT w - y ">"
R A H U L

User avatar
GMAT Instructor
Posts: 15539
Joined: Tue May 25, 2010 12:04 pm
Location: New York, NY
Thanked: 13060 times
Followed by:1906 members
GMAT Score:790

by GMATGuruNY » Wed Oct 02, 2013 2:28 am
theCodeToGMAT wrote:If w + x < 0, is w - y > 0?
(1) x + y < 0
(2) y < x < w

My Steps to Solutions:
w + x < 0
w + x < y - y
w - y < - y - x
w - y < - (x + y)
The reasoning above is valid.
Statement 1 says x + y < 0 ... So, w - y > 0 ... Sufficient
This reasoning is not valid.
Since x+y < 0, -(x+y) > 0.
Since -(x+y) is POSITIVE, w-y < - (x + y) can be rephrased as follows:
w-y < POSITIVE.

This tells us nothing about the relationship between w and y.
It's possible that w<y, that w=y, or that w>y.

To illustrate:
If x+y = -10, then -(x+y) = 10.
In this case, w-y < 10.
Here, it's possible that w=1 and y=2, that w=1 and y=1, or that w=2 and y=1.

Thus, statement 1 is INSUFFICIENT.
Private tutor exclusively for the GMAT and GRE, with over 20 years of experience.
Followed here and elsewhere by over 1900 test-takers.
I have worked with students based in the US, Australia, Taiwan, China, Tajikistan, Kuwait, Saudi Arabia -- a long list of countries.
My students have been admitted to HBS, CBS, Tuck, Yale, Stern, Fuqua -- a long list of top programs.

As a tutor, I don't simply teach you how I would approach problems.
I unlock the best way for YOU to solve problems.

For more information, please email me (Mitch Hunt) at [email protected].
Student Review #1
Student Review #2
Student Review #3

Legendary Member
Posts: 641
Joined: Tue Feb 14, 2012 3:52 pm
Thanked: 11 times
Followed by:8 members

by gmattesttaker2 » Wed Oct 02, 2013 4:30 pm
Hello,

I was just wondering why this would be wrong since I thought that we can add/subtract in-equalites as long as they have the same sign:

If w + x < 0, is w - y > 0?
(1) x + y < 0

So, w + x < 0
& y + x < 0

So, w - y < 0


Thanks,
Sri

Master | Next Rank: 500 Posts
Posts: 391
Joined: Sat Mar 02, 2013 5:13 am
Thanked: 50 times
Followed by:4 members

by rakeshd347 » Wed Oct 02, 2013 6:26 pm
gmattesttaker2 wrote:Hello,

I was just wondering why this would be wrong since I thought that we can add/subtract in-equalites as long as they have the same sign:

If w + x < 0, is w - y > 0?
(1) x + y < 0

So, w + x < 0
& y + x < 0

So, w - y < 0


Thanks,
Sri
Hi Sri,

Have a look at my explanation above and you will understand.

GMAT/MBA Expert

User avatar
GMAT Instructor
Posts: 768
Joined: Wed Dec 28, 2011 4:18 pm
Location: Berkeley, CA
Thanked: 387 times
Followed by:140 members

by Mike@Magoosh » Wed Oct 02, 2013 7:32 pm
gmattesttaker2 wrote:Hello,

I was wondering why this would be wrong since I thought that we can add/subtract inequalites as long as they have the same sign:

If w + x < 0, is w - y > 0?
(1) x + y < 0

So, w + x < 0
& y + x < 0

So, w - y < 0

Thanks,
Sri
Sri,
I'm responding to your private message. Sometimes the best way to think about what's true for inequalities is to think in terms of ordinary numbers.

Inequality A: 15 < 17
Inequality B: 3 < 100
Both true. Both are in the same direction.

If we add them, all is well,
A + B: 18 < 117

BUT, if we subtract them ....
A - B: 12 > -83
Here, the direction of inequality reverses when we subtract (A - B). Notice that the direction wouldn't reverse if we subtracted (B - A). There's no predictable pattern --- subtracting inequalities is trouble!

It's NEVER a good idea to subtract two inequalities. Again, adding them is perfectly legit. You are also allowed to add or subtract any constant to both sides, and the inequality would still work. BUT, subtracting one inequality from another is just asking for big problems. That's what's so tempting about this brilliantly designed DS ---- it just lures the unsuspecting test taker into making a very predictable mistake. It's so good for you to learn about these predictable mistakes, so you can learn to avoid them.

Does all this make sense?
Mike :-)
Magoosh GMAT Instructor
https://gmat.magoosh.com/

Master | Next Rank: 500 Posts
Posts: 391
Joined: Sat Mar 02, 2013 5:13 am
Thanked: 50 times
Followed by:4 members

by rakeshd347 » Wed Oct 02, 2013 8:01 pm
gmattesttaker2 wrote:Hello,

I was just wondering why this would be wrong since I thought that we can add/subtract in-equalites as long as they have the same sign:

If w + x < 0, is w - y > 0?
(1) x + y < 0

So, w + x < 0
& y + x < 0

So, w - y < 0


Thanks,
Sri
Hi Sri,

Here is the golden rule of inequality:

A>C and B>D then you can add both inequality: A+B>C+D.
However you can't subtract both inequality A-B>C-D....this is wrong.
If you want to subtract then you have to have the opposite sign.

Let me explain you here:

A>C and B>D...or we can say A>C and D<B....one and same thing right.
Now you can subtract A-D>C-B ( now the main point here to note is inequality sign will take the sign of the inequality from which we are subtracting)

A>C and D<B now suppose you want to get D-A<B-C....in this case it will take the sign of D inequality because you are subtracting from D. In the above inequality you were subtracting from A.

Suppose
3>2 and 3>1...now if you subtract 3-3>2-1 or 0>1 no this is wrong
but if you were to do.

3>2 and 1<3 then you can do 3-1>2-3 which is 2>-1

This Rule will always hold true. Not many people know it but this is a good tool to tackle inequality questions.